Hello fellow mathematicians,
This upcoming weekend marks the inaugural Proofathon contest. Essentially, Proofathon is a USAMO-like, online math contest administered over the course of two days, eight problems. Show your support and compete in the inaugural Proofathon! Also, note that if we get over 200 likes on our Facebook page, we will have a special surprise.
Until then, I'll leave you with one of our shortlisted problems:
Show that none of the elements in the set is a perfect cube.
Easy Math Editor
This discussion board is a place to discuss our Daily Challenges and the math and science related to those challenges. Explanations are more than just a solution — they should explain the steps and thinking strategies that you used to obtain the solution. Comments should further the discussion of math and science.
When posting on Brilliant:
*italics*
or_italics_
**bold**
or__bold__
paragraph 1
paragraph 2
[example link](https://brilliant.org)
> This is a quote
\(
...\)
or\[
...\]
to ensure proper formatting.2 \times 3
2^{34}
a_{i-1}
\frac{2}{3}
\sqrt{2}
\sum_{i=1}^3
\sin \theta
\boxed{123}
Comments
Solution: Cubes are always −1,0,1(mod9), and since the given numbers are all 4(mod9), there are no cubes in the set. ■
Log in to reply
How many perfect fifth powers can be written in the form of 9⋅2k−4?
Log in to reply
Let m5=9⋅2k+6. From this, we see that 3∣m5⟹3∣m, and so 9∣m5. However, 9⋅2k+6≡6(mod9), a contradiction. Therefore, the answer is 0.
This can also be found by writing out the quintic residues modulo 9.
Log in to reply
9⋅2k−4 Sorry!
I messed up (addition error):Log in to reply
m5=9⋅2k−4.
LetIf k<5, then we can check manually: k=0⟹m5=5, no solutionk=1⟹m5=14, no solutionk=2⟹m5=32, m=2k=3⟹m5=68, no solutionk=4⟹m5=140, no solution
Otherwise, if k>4, then 2∣9⋅2k−4⟹2∣m. Then, 32∣9⋅2k−4. Hence, 32∣9⋅32⋅2k−5−4, which is not possible.
Therefore, the answer is 1. ■
Log in to reply
You can always search for Proofathon in facebook :) P.S.-The link is working fine for me.
Is there some easy way to figure out that the numbers are all congruent to 4(mod9)? This is how I figured it:
Considered modulo 9, the terms are 4∗10k for all k≥1. It can be shown that the numbers 36,396,3996,39996,399....96 are all multiples of 9, namely they are 9∗44...4, so thus the numbers 40,400,4000,40000... are congruent to 4(mod9).
Log in to reply
More simply, 4⋅10k+9≡4⋅1k+9≡4(mod9)
Let S(n) denote the sum of the digits of n. Then, n≡S(n)(mod9) (this is not hard to prove).
If anything, look at our logo!
how do I submit the problems??is there any registration??
Log in to reply
Hey Jawwad,
If you want to submit original problems for consideration for use on future contests, you can email us at Proofathon@gmail.com. If you are asking where to submit solutions to the problems for the contests, visit our contest page http://proofathon.org/Pages/contests.php on the date of the contest. The first contest will be held this weekend from October 26 at 12:00 AM EDT to October 27 at 11:59 EDT.
To register, simply create an account at http://proofathon.org/Pages/account.php and log on with that account on the contest date.
Note that the date has changed, and Proofathon is starting this Friday at midnight (which is in 4 hours).
The contest is ongoing. One can find the problems here.